Difference between revisions of "1990 USAMO Problems/Problem 5"
Vallannion (talk | contribs) (→Solution) |
Danielguo94 (talk | contribs) (→Solution) |
||
Line 3: | Line 3: | ||
== Solution == | == Solution == | ||
− | Let <math>A'</math> be the intersection of the two circles. <math>AA'</math> is perpendicular to both <math>BA'</math>, <math>CA'</math> implying <math>B</math>, <math>C</math>, <math>A'</math> are collinear. Since <math>A'</math> is the foot of the altitude from <math>A</math>: <math>A</math>, <math>H</math>, <math>A'</math> are concurrent, where <math>H</math> is the orthocentre. | + | Let <math>A'</math> be the intersection of the two circles (other than <math>A</math>). <math>AA'</math> is perpendicular to both <math>BA'</math>, <math>CA'</math> implying <math>B</math>, <math>C</math>, <math>A'</math> are collinear. Since <math>A'</math> is the foot of the altitude from <math>A</math>: <math>A</math>, <math>H</math>, <math>A'</math> are concurrent, where <math>H</math> is the orthocentre. |
Now, <math>H</math> is also the intersection of <math>BB'</math>, <math>CC'</math> which means that <math>AA'</math>, <math>MN</math>, <math>PQ</math> are concurrent. Since <math>A</math>, <math>M</math>, <math>N</math>, <math>A'</math> and <math>A</math>, <math>P</math>, <math>Q</math>, <math>A'</math> are cyclic, <math>M</math>, <math>N</math>, <math>P</math>, <math>Q</math> are cyclic by the radical axis theorem. | Now, <math>H</math> is also the intersection of <math>BB'</math>, <math>CC'</math> which means that <math>AA'</math>, <math>MN</math>, <math>PQ</math> are concurrent. Since <math>A</math>, <math>M</math>, <math>N</math>, <math>A'</math> and <math>A</math>, <math>P</math>, <math>Q</math>, <math>A'</math> are cyclic, <math>M</math>, <math>N</math>, <math>P</math>, <math>Q</math> are cyclic by the radical axis theorem. |
Revision as of 20:29, 18 August 2012
Problem
An acute-angled triangle is given in the plane. The circle with diameter intersects altitude and its extension at points and , and the circle with diameter intersects altitude and its extensions at and . Prove that the points lie on a common circle.
Solution
Let be the intersection of the two circles (other than ). is perpendicular to both , implying , , are collinear. Since is the foot of the altitude from : , , are concurrent, where is the orthocentre.
Now, is also the intersection of , which means that , , are concurrent. Since , , , and , , , are cyclic, , , , are cyclic by the radical axis theorem.
Alternate solutions are always welcome. If you have a different, elegant solution to this problem, please add it to this page.
Resources
1990 USAMO (Problems • Resources) | ||
Preceded by Problem 4 |
Followed by Final Question | |
1 • 2 • 3 • 4 • 5 | ||
All USAMO Problems and Solutions |